PT54.S2.Q18 - columnist: taking a strong position on

emilycyoung1emilycyoung1 Free Trial Member
edited September 2016 in Logical Reasoning 234 karma
Question 18 talks about the columnist and taking a strong position. I have had several people try to diagram this in conditional statements and I still don't understand 1. my task and 2. how to diagram this in conditionals, and how to get to the answer choice C. Please help
https://7sage.com/lsat_explanations/lsat-54-section-2-question-18/

Comments

  • nye8870nye8870 Alum
    1749 karma
    @amipp170 wrote this in the comments. I think it is well stated. See if this helps. If not we'll delve deeper.
    "taking a strong position on an issue makes one likely to misinterpret or ignore additional evidence that conflicts with ones stand. fair enough.
    in order to understand an issue fully, it is essential to consider such evidence impartially.

    UIF→CEI
    _
    if one has not already considered all important evidence conflicting with that position, then its best not to take a strong position.

    /CIE→/SP

    SP→CIE

    i think its safe to say that CEI and CIE are the same thing: that considering evidence imperially is the same as considering all important evidence that conflicts with the position.

    so from here take it as a SA question, SP→UIF. if you take a strong position, you must understand an issue fully.

    a) this reverses the logic.
    b) this gives us the necessary condition for CEI. we dont know that.
    c) this is the contrapositive of what i have. perfect.
    d) we dont have a sufficient condition that goes to UIF (unless you say /SP but this ac doesnt say that)
    e) no only if one has considered all important evidence conflicting with that position."
  • emilycyoung1emilycyoung1 Free Trial Member
    234 karma
    @nye8870 i just don't understand how we get from sp to uif. I have seen this multiple times with conditionals and it may be something I just never knew. Where its something like this: A---> B
    C---->B then the "gap" is A----> C. I don't understand how to get there.
  • PositivePositive Alum Member
    edited September 2016 426 karma
    @emilycyoung1 I think you are confused. "A-->C" should be "C-->A"

    Question:
    P1: A-->B
    Conclusion: C-->B
    what do we need?
    P2: C-->A

    which is the basic conditional logic form of this question.
    Please correct me if I'm wrong!
  • nye8870nye8870 Alum
    edited June 2018 1749 karma
    @"Kings Never Die" said:
    "A-->C" should be "C-->A"
    yes
  • nye8870nye8870 Alum
    1749 karma
    So the premise asserts A --> B. The conclusion asserts C --> B. So now lets make a statement: If all C's are A's (this is the missing premise) then that looks like this C --> A .... add this to chain up A --> B (the A's overlap and you get a chain looks like this C --> A --> B which means All C's are B's (the conclusion)
  • PearsonSpecterLittUpPearsonSpecterLittUp Alum Member
    588 karma

    Bringing this back cause I took this PT yesterday and found this helpful

Sign In or Register to comment.